PLEASE HELP MARKING BRAINLEIST JUST ANSWER ASAP

PLEASE HELP MARKING BRAINLEIST JUST ANSWER ASAP

Answers

Answer 1

Answer:

3c + 19

Step-by-step explanation:

Perimeter: P = a + b + c

P = (c + 10) + (c + 6) + (c + 3) = 3c + 19


Related Questions

camilia saved 4/5% of her allowance. What is this percent expressed as a fraction and as a decimal

Answers

80% is the percentage, and .8 is the decimal

9) A survey asked 915 people how many times per week they dine out at a restaurant. Th results are presented in the following table. Number of Times Frequency 134 273 249 127 72 30 24 Total915 Consider the 915 people to be a population. Let X be the number of times per week a person dines out for a person sampled at random from this population. Compute the standard deviation A) 2.1 в) 1.5 C 1.9 D) 2.2 10) Determine the indicated probability for a binomial experiment with the given number of trials n and the given success probability p 15, p 0.4, P(12) A) 0.0016 B) 0.0634 C 0 4000 D) 0.0000

Answers

The correct option is B) 0.0634.

1) Standard deviationThe table is shown as below:Number of Times Frequency 1 34 2 73 3 249 4 127 5 72 6 30 7 24 Total 915The given population contains 915 people. Let X be the number of times per week a person dines out for a person sampled at random from this population.The population standard deviation is given by the formula below:$$\sigma = \sqrt{\frac{\sum (X-\mu)^2}{N}}$$where N is the population size, X is the value of the individual element of the population, μ is the mean value of the population. $$\sigma = \sqrt{\frac{\sum (X-\mu)^2}{N}}$$$$= \sqrt{\frac{\sum X^2 - 2X\mu + \mu^2}{N}}$$We need to compute the variance first.$$\sigma^2 = \frac{\sum X^2 - 2X\mu + \mu^2}{N}$$$$= \frac{(1^2 \cdot 34 + 2^2 \cdot 73 + 3^2 \cdot 249 + 4^2 \cdot 127 + 5^2 \cdot 72 + 6^2 \cdot 30 + 7^2 \cdot 24) - 2\cdot 3.176 \cdot (34 + 2\cdot 73 + 3\cdot 249 + 4\cdot 127 + 5\cdot 72 + 6\cdot 30 + 7\cdot 24) + 3.176^2 \cdot 915}{915}$$$$= 3.5689$$Therefore, the population standard deviation is given by:$\sigma = \sqrt{3.5689} = 1.8911\approx 1.9$Hence, the correct option is C) 1.92) The probability of a binomial experiment can be calculated by the formula below:$${\rm P}(X=k) = \binom{n}{k}p^k(1-p)^{n-k}$$where n is the number of trials, k is the number of successes, p is the probability of success for each trial, and $1-p$ is the probability of failure for each trial.Here, n = 15 and p = 0.4.We need to find the probability of getting 12 successes out of 15 trials. Hence, k = 12.Using the formula above, we get$${\rm P}(X=12) = \binom{15}{12}0.4^{12}0.6^{3}$$$$= \frac{15 \times 14 \times 13}{3 \times 2 \times 1} \cdot 0.4^{12} \cdot 0.6^{3}$$$$= 0.0633605376$$Hence, the correct option is B) 0.0634.

Learn more about Option

brainly.com/question/22100121

#SPJ11

let x1 and x2 be two independent random variables both with mean 10 and variance 5. let y 2x1 x2 3 2. find the mean and the variance of y.

Answers

As a result, y has a mean of 203 and a variance of 85 as let x1 and x2 be two independent random variables both with mean 10 and variance 5.

what is variable ?

A variable is a symbol or letter that is used to indicate a variable quantity in mathematics. The context or issue under consideration can alter the value of a variable. In order to express relationships between quantities, variables are frequently utilized in equations, formulae, and functions. For instance, x and y are variables in the equation y = mx + b, which depicts the linear relationship between x and y. Variables in statistics can reflect various traits or features of a population or sample, such as age, body mass index, or income.

given

To get the mean and variance of y, we can apply the characteristics of expected value and variance:

We can start by determining the expected value of y:

E[y] = 2E[x1] = E[2x1x2 + 3x1 + 2]

By the linearity of expectation, E[x2] + 3E[x1] + 2 is 2(10)(10) + 3(10) + 2 = 203.

Next, we may determine y's variance:

Var(y) = Var(3x1 + 2 + 2 + 3x1 ) = 4

Var(x1)

Var(x2) + 9

Var(x1) + Var(constant) = 4(5)(5) + 9(5) + 0 = 85 since x1 and x2 are independent.

As a result, y has a mean of 203 and a variance of 85 as let x1 and x2 be two independent random variables both with mean 10 and variance 5.

To know more about variable visit :-

https://brainly.com/question/2466865

#SPJ1

The segment of a circle of radius 14 cm has an angle of 135° at the centre. Calculate its perimeter.​

Answers

Answer: 61

Step-by-step explanation:

explination in image

HELPPPP HURRY PLSS………………..

Answers

Answer:

C is your answer

Step-by-step explanation:

in my opinion, i think it would be the mode.

fifty-three percent of all persons in the u.s. population have at least some college education. choose 10 persons at random. what is the probability that exactly one-half have some college education?

Answers

The probability that exactly one-half have some college education is 0.0439 (rounded to four decimal places)

Step by step explanation: We can use the binomial probability distribution formula for this problem, where:p = probability of success (i.e., having some college education) = 0.53q = probability of failure (i.e., not having some college education) = 0.47n = number of trials (i.e., persons chosen at random) = 10x = number of successes (i.e., persons with some college education) = [tex]5P(x = 5) = C(10,5)p^5q^5[/tex] where [tex]C(n,r)[/tex] is the combination function that gives the number of ways to choose r items from a set of n items.

It is given by:[tex]C(n,r) = n! / (r!(n-r)!)[/tex] Thus, we can substitute the given values and compute:[tex]P(x = 5) = C(10,5)p^5q^5 = 252(0.53)^5(0.47)^5= 0.0439[/tex] (rounded to four decimal places)Therefore, the probability that exactly one-half have some college education is 0.0439.

See more about binomial probability at: https://brainly.com/question/24756209

#SPJ11

Simplify the product: (4x-3) (3x+1)
O A. x-2
OB. 12x²-3
O C. 12x-5x-3
OD. 12x³-9x² +4x-3

Answers

Answer:

[tex]\large\boxed{\mathtt{C) \ 12x^{2}-5x-3}}[/tex]

Step-by-step explanation:

[tex]\textsf{For this problem, we are asked to multiply 2 Binomials.}[/tex]

[tex]\textsf{We should use the FOIL Method.}[/tex]

[tex]\large\underline{\textsf{FOIL Broken Down;}}[/tex]

[tex]\textsf{F - Fronts}[/tex]

[tex]\textsf{O - Outers}[/tex]

[tex]\textsf{I - Inners}[/tex]

[tex]\textsf{L - Last}[/tex]

[tex]\large\underline{\textsf{For Example.}}[/tex]

[tex]\mathtt{(2x+2y)(3x+3y)}[/tex]

[tex]\mathtt{(F) \ 2x \times 3x = 6x^{2}}[/tex]

[tex]\mathtt{(O) \ 2x \times 3y = 6xy}[/tex]

[tex]\mathtt{(I) \ 2y \times 3x = 6xy}[/tex]

[tex]\mathtt{(L) \ 2y \times 3y = 6y^{2}}[/tex]

[tex]\textsf{Let's use the FOIL Method for our problem.}[/tex]

[tex]\large\underline{\textsf{FOIL;}}[/tex]

[tex]\mathtt{(4x-3)(3x+1)}[/tex]

[tex]\mathtt{(F) \ 4x \times 3x = 12x^{2}}[/tex]

[tex]\mathtt{(O) \ 4x \times 1 = 4x}[/tex]

[tex]\mathtt{(I) \ -3 \times 3x = -9x}[/tex]

[tex]\mathtt{(L) \ -3 \times 1 = -3}[/tex]

[tex]\large\underline{\textsf{We should have;}}[/tex]

[tex]\mathtt{12x^{2}+4x-9x-3}[/tex]

[tex]\large\underline{\textsf{Combine Like Terms;}}[/tex]

[tex]\large\boxed{\mathtt{C) \ 12x^{2}-5x-3}}[/tex]

Q4 NEED HELP PLEASE HELP

Answers

Answer:

D. The electrician charges $23 per hour.

Step-by-step explanation:

C(h)= 23h+30 is in the form y=mx +b

$30 is the initial fee (b)

$23 is the amount charged per hour (h)

The area of a rectangle is x2 – 6x +8. Find its possible length and breadth:

Answers

Answer:

Step-by-step explanation:

To find the possible length and breadth of the rectangle, we need to factor the given expression:

x^2 - 6x + 8 = (x - 4)(x - 2)

Therefore, the length and breadth of the rectangle can be any combination of (x-4) and (x-2).

For example, if we choose (x-4) as the length and (x-2) as the breadth, we have:

Length = x - 4

Breadth = x - 2

Conversely, if we choose (x-2) as the length and (x-4) as the breadth, we have:

Length = x - 2

Breadth = x - 4

So, the possible length and breadth of the rectangle are (x-4) and (x-2), and vice versa.

Answer:(x-4) and (x-2)

Step-by-step explanation:

solve the following questions based on the information provided: i. six students a, b, c, d, e and f participated in a self-evaluation test of quants and data interpretation (d.i.). ii. the total marks of a in quants was just above c and in d.i. just above f. iii. b was just above c in d.i. but he scored less than d in quants. iv. f got more marks than d and e in d.i. but did not perform as well in quants as in d.i. as compared to d and e. v. no one is in between c and d in quant and c and a in d.i. 16.who got the highest marks in d.i.? a b c data inadequate 17.which of the following students has scored the least in d.i.? only d only e only d or e none of these 18.who was just below d in quants? b e c data inadequate 19.which of the given statements is not necessary to answer the questions? (ii) (iii) (iv) all are necessary

Answers

For all the following questions all the statements are required, It is possible to determine that c was just below d in quants based on the given information. Additionally, all given statements (ii), (iii), and (iv) are necessary to answer the questions, as they provide essential information about the relative performance of the students in quants and d.

The given statements which are not necessary to answer the questions are:

   To answer this question, we need to find out who got the highest marks in d.i. According to statement (iv), F got more marks than D and E in d.i. Therefore, we can conclude that F got the highest marks in d.i.    To answer this question, we need to find out who scored the least in d.i. However, the information provided does not tell us the score of any student in d.i., except that F got the highest marks. Therefore, we cannot determine which student scored the least in d.i.    To answer this question, we need to find out who was just below D in quants. According to statement (v), no one is in between C and D in quants, which means that D got the highest marks in quants among the students listed. According to statement (ii), A's marks in quants were just above C's. Therefore, E must have been just below D in quants.   To answer this question, we need to identify the statement that is not necessary to answer the questions. Statements (ii) and (iv) provide information about who got more marks in d.i. than certain other students, which is necessary to answer question 16. Statement (v) provides information about the order of marks in quants, which is necessary to answer question 18. Therefore, the statement that is not necessary to answer the questions is statement (iii), which only provides information about B's marks in quants relative to D and in d.i. relative to C. This information is not required to answer any of the questions.

All of the other statements (II, III, IV) are necessary to answer the questions.

To learn more about the data interpretation:

https://brainly.com/question/1638242.

#SPJ11

What is the value of x in √1+ 25/144 =1+ /12 ?

Answers

The solution to the equation √(1 + 25/144) = 1 + x/12 is x = 5/6. This was achieved by simplifying the left side of the equation and isolating x on one side.

To solve the equation √(1 + 25/144) = 1 + x/12, we start by simplifying the left side of the equation. The expression inside the square root can be simplified to (144 + 25)/144 = 169/144. Taking the square root of this fraction gives us √(169/144) = (13/12).

Next, we subtract 1 from both sides of the equation to isolate x on one side: (13/12) - 1 = x/12. This simplifies to 1/12 = x/12.

Finally, we multiply both sides by 12 to solve for x: x = (1/12)*12 = 5/6.

So the solution to the equation √(1 + 25/144) = 1 + x/12 is x = 5/6.

Learn more about algebra here: brainly.com/question/24875240

#SPJ4

A triangle can have sides with the following measures: 1, 1, 2
True
False

Answers

Answer: false

Step-by-step explanation:

Arrange the steps to find an inverse of a modulo m for each of the following pairs of relatively prime integers using the Euclidean algorithm in the order. a = 2, m=17 Rank the options below. The ged in terms of 2 and 17 is written as 1 = 17-8.2. By using the Euclidean algorithm, 17 = 8.2 +1. The coefficient of 2 is same as 9 modulo 17. 9 is an inverse of 2 modulo 17. The Bézout coefficients of 17 and 2 are 1 and 8, respectively. a = 34, m= 89 Rank the options below. The steps to find ged(34,89) = 1 using the Euclidean algorithm is as follows. 89 = 2.34 + 21 34 = 21 + 13 21 = 13 + 8 13 = 8 + 5 8 = 5 + 3 5 = 3 + 2 3 = 2+1 Let 34s + 890= 1, where sis the inverse of 34 modulo 89. $=-34, so an inverse of 34 modulo 89 is -34, which can also be written as 55. The ged in terms of 34 and 89 is written as 1 = 3 - 2 = 3-(5-3) = 2.3-5 = 2. (8-5)- 5 = 2.8-3.5 = 2.8-3. (13-8)= 5.8-3.13 = 5. (21-13)-3.13 = 5.21-8. 13 = 5.21-8. (34-21) = 1321-8.34 = 13. (89-2.34) - 8.34 = 13.89-34. 34 a = 200, m= 1001 Rank the options below. By using the Euclidean algorithm, 1001 = 5.200 +1. Let 200s + 1001t= 1, where sis an inverse of 200 modulo 1001. The ged in terms of 1001 and 200 is written as 1 = 1001 - 5.200. s=-5, so an inverse of 200 modulo 1001 is -5.

Answers

We have that, using Euclid's algorithm, we find the inverse of 200 modules 1001 is -5 (or 1001+5).

How do we find the inverse of a modulus?

To find the inverse of a module m using Euclid's algorithm, the steps are as follows:

1. Calculate the greatest common divisor (GCD) of a and m using the Euclidean algorithm.

2. Let a = GCD * s + m*t, where s is the inverse of a module m.

3. The GCD in terms of a and my is written as 1 = m-s*a.

4. Find s = -a, so the inverse of a module m is -a (or m+s).

For example, a = 2, m=17, so GCD = 1 = 17-8*2 and the inverse of 2 modulo 17 is -8 (or 17+8). Similarly, for a = 34, m= 89, the GCD = 1 = 89-34*2 and the inverse of 34 modulo 89 is -34 (or 89+34). Finally, for a = 200, m= 1001, the GCD = 1 = 1001-5*200 and the inverse of 200 modulo 1001 is -5 (or 1001+5).

See more information about Euclid's algorithm at: https://brainly.com/question/24836675

#SPJ11

Shanika bought 1 Notebook and 5 pencils at a cost of $10.00. Jules paid $14.90 for 1 Notebook and 12 pencils. How much did Jules pay for 1 pencil if both pay the similar prices?​

Answers

ANSWER

Jules paid $0.70 for one pencil

STEPS

Cost of One Pencil.

Let's assume the cost of one notebook be x and one pencil be y.

According to the given information,

Shanika bought 1 notebook and 5 pencils at a cost of $10.00. Therefore,

1x + 5y = 10 --- equation 1

Jules paid $14.90 for 1 Notebook and 12 pencils. Therefore,

1x + 12y = 14.9 --- equation 2

We need to find the cost of one pencil, i.e., y.

We can solve the above equations simultaneously to find the value of y.

Multiplying equation 1 by 12 and equation 2 by 5, we get:

12x + 60y = 120 --- equation 3

5x + 60y = 74.5 --- equation 4

Subtracting equation 4 from equation 3, we get:

7x = 45.5

x = 6.5

Substituting the value of x in equation 1, we get:

1(6.5) + 5y = 10

5y = 3.5

y = 0.7

Therefore, Jules paid $0.70 for one pencil.

ChatGPT

Answer:

Jules pay:

$0.7

Step-by-step explanation:

1n + 5p = 10                  Eq. 1

1n + 12p = 14.9              Eq. 2

n = cost of one notebook

p = cost of one pencil

From Eq. 1

n = 10 - 5p               Eq. 3

From Eq. 2

n = 14.9 - 12p            Eq. 4

Equalizing Eq. 3 and Eq. 4:

10 - 5p = 14.9 - 12p

12p - 5p = 14.9 - 10

7p = 4.9

p = 4.9 / 7

p = 0.7

From Eq. 3:

n = 10 - 5p

n = 10 - 5*0.7

n = 10 - 3.5

n = 6.5

Check:

From Eq. 2:

n + 12p = 14.9

6.5 + 12*0.7 = 14.9

6.5 + 8.4 = 14.9

Then:

1 pencil = $0.7

Let $ABCD$ be a parallelogram. Extend $\overline{DA}$ through $A$ to a point $P,$ and let $\overline{PC}$ meet $\overline{AB}$ at $Q$ and $\overline{DB}$ at $R.$ Given that $PQ = 735$ and $QR = 112,$ find $RC.$

Answers

The x ≤ 32Putting x = 24 in the expression we get RC = 96Therefore, the value of RC is 96.

In order to find RC, we will make use of the given information in the following manner: Given that ABDC is a parallelogram. Hence, AB = DC. We have also been given that PQ = 735 and QR = 112.Now, extend PQ to meet DC at S.Let PS = x; then DS = DC - x = AB - x. (Since AB = DC)We have that PS/SP = QR/RB (Since PQR is similar to DBR)Therefore, we getx/SP = 112/(AB - x)We can cross multiply and simplify to getSP = (112* x)/(AB - x)......(1)Further, we have that AQ/QB = SP/BR (Since PQR is similar to AQB)Therefore, we getx/(AB - x) = SP/BROn substituting the value of SP from equation (1) above, we getx/(AB - x) = (112* x)/(BR*(AB - x))Therefore, we getBR = (x*(AB - x)*QR)/[PQ*(AB - 2*x)]BR = (x*(32 - x)*112)/(735*(32 - 2*x))BR = (56*x*(16 - x))/(245*(16 - x))BR = (56*x/245)Since the sum of all sides of a parallelogram is equal to the sum of its opposite sides, we have thatRC + QR = AB + AQ - QBRearranging the terms we getRC = AB + AQ - QB - QR......(2)Now, AQ = PQ - APSubstituting the values of PQ and AP we get AQ = 735 - (DC - x) = 735 - 32 + x = x + 703Also, QB = AB - AQ = (32 - x) - (x + 703) = -x - 671Substituting the values of AQ and QB in equation (2) above we getRC = 32 + (x + 703) + (x + 671) - 112RC = 48 + 2xRC = 2(x + 24)We know that AB = 32, hence, x ≤ 32Putting x = 24 in the expression we get RC = 96Therefore, the value of RC is 96.

Learn more about Expression

brainly.com/question/23246712

#SPJ11

I need help! I need the graph drawn and the steps to how I got the answer but I don’t know it! Please help me!

Answers

Answer:

25 computers per hour

Step-by-step explanation:

look ate the point 2 hours corresponding to 50 computers

50/2 = 25/1 or 25 computers per hour

Victor spent $61 on some sandpaper for his model
cars. He bought 2 packages of the smallest-grain
sandpaper and spent the rest on the largest-grain
sandpaper. How many packages of the largest-
grain sandpaper did he buy?
Size of
Grain (cm)
0.003
0.011
0.001
Cost per
Package (S)
13
7
20

Answers

Answer:

3 packages

Step-by-step explanation:

61 - 20 * 2 (small grain sandpaper) = $21 remain. The largest grain costs $7, so 21/7 = 3 packages of the largest grain sandpaper was bought.

Hope this helped!

Can some one help me? It’s three parts but the questions states use the interval notation to write the intervals over which f is (a) increasing, (b) decreasing, and (c) constant. The last question also says topics related to if its constant or not.

Answers

The function is constant from approximately x = -4 to x = -3 and from approximately x = -1 to x = 1. So, the constant intervals are (-4, -3) and (-1, 1)

What exactly are function and example?

A function, which produces one output from a single input, is an illustration of a rule. The picture was obtained from Alex Federspiel. The equation y=x2 serves as an example of this.

a) We can see that the function is increasing from approximately x = -3 to x = -1 and from approximately x = 1 to x = 2.5. So, the increasing intervals are (-3,-1) and (1, 2.5)

(b) We can see that the function is decreasing from approximately x = -2 to x = -0.5 and from approximately x = 3 to x = 4. So, the decreasing intervals are (-2, -0.5) and (3, 4)

(c) We can see that the function is constant from approximately x = -4 to x = -3 and from approximately x = -1 to x = 1. So, the constant intervals are (-4, -3) and (-1, 1)

To know more about function visit:-

https://brainly.com/question/12431044

#SPJ1

11. How much time will it take for ₹5000
5618 at 6% per annum
annually?
to become
compounded

Answers

Answer:

2.31 Years

Step-by-step explanation:

To calculate the time it will take for ₹5000 to grow to ₹5618 with a 6% annual interest rate when compounded annually, we can use the following formula:

A = P(1 + r/n)^(nt)

Where:

A = the final amount (₹5618)

P = the principal amount (₹5000)

r = the annual interest rate (6% or 0.06)

n = the number of times the interest is compounded per year (1, since it's compounded annually)

t = the time period in years

Plugging in the values, we get:

5618 = 5000(1 + 0.06/1)^(1t)

Simplifying:

1.1236 = 1.06^t

Taking the natural logarithm of both sides:

ln(1.1236) = ln(1.06^t)

Using the power rule of logarithms:

ln(1.1236) = t ln(1.06)

Solving for t:

t = ln(1.1236) / ln(1.06)

t ≈ 2.31 years

Therefore, it will take approximately 2.31 years for ₹5000 to grow to ₹5618 at a 6% annual interest rate when compounded annually.

Solve 2log 12 (-8x)=6

Answers

The solution to the logarithmic equation [tex]2log12(-8x) = 6[/tex] is [tex]x = -9/32[/tex] .

What are logarithmic properties ?

Logarithmic properties are the rules that govern the behavior of logarithmic functions. These properties are important in simplifying logarithmic expressions and solving logarithmic equations. Some of the commonly used logarithmic properties include:

Product property: [tex]logb(xy) = logb(x) + logb(y)[/tex]

This property allows us to simplify the logarithm of a product of two numbers into the sum of logarithms of the individual numbers.

Quotient property: [tex]logb(x/y) = logb(x) - logb(y)[/tex]

This property allows us to simplify the logarithm of a quotient of two numbers into the difference of logarithms of the individual numbers.

Power property:[tex]logb(x^y) = ylogb(x)[/tex]

This property allows us to simplify the logarithm of a power of a number by bringing the exponent outside of the logarithm and multiplying it with the logarithm of the base.

Change of base formula: [tex]logb(x) = logc(x) / logc(b)[/tex]

This property allows us to change the base of a logarithm by dividing the logarithm of the number by the logarithm of the base in a different base.


Solving the given logarithmic equation :

The equation can be solved by using logarithmic properties and basic algebraic manipulation.

We can begin by using the property that states [tex]loga(b^n) = nloga(b)[/tex] for any base a and any positive real number b. Applying this property, we can rewrite the left side of the equation as:

[tex]log12((-8x)^2) = log12(64x^2)[/tex]

Next, we can use the property that states [tex]loga(b) = c[/tex] is equivalent to [tex]a^c = b[/tex]. Applying this property, we can rewrite the equation as:

[tex]12^{2log12(64x^2)} = 12^6[/tex]

Simplifying the left side, we get:

[tex]64x^2 = 12^6 / 12^2[/tex]

[tex]64x^2 = 144[/tex]

Dividing both sides by 64, we get:

[tex]x^2 = 144/64[/tex]

[tex]x^2 = 9/4[/tex]

Taking the square root of both sides, we get:

[tex]x=\pm 3/2[/tex]

However, we need to check the solutions for extraneous roots since the original equation has a logarithm with a negative argument. We can see that the solution x = 3/2 is extraneous since it results in a negative argument for the logarithm. Therefore, the only valid solution is x = -9/32.

To know more about logarithmic equation visit :

brainly.com/question/30085872

#SPJ1

a cylindrical glass is half full of lemonade. the ratio of lemon juice to water in the lemonade is $1:11$. if the glass is $6$ inches tall and has a diameter of $2$ inches, what is the volume of lemon juice in the glass? express your answer as a decimal to the nearest hundredth.

Answers

The volume of lemon juice in the glass is 0.38 cubic inches.

Explanation:

Given,

Diameter of cylindrical glass = 2 inchesHeight of cylindrical glass = 6 inchesRatio of lemon juice to water in the lemonade = 1:11

Let the volume of the lemonade in the glass be V cubic inches

Therefore, the volume of lemon juice in the lemonade is [tex]$\frac{1}{12}$[/tex] V cubic inches

Volume of water in the lemonade is [tex]$\frac{11}{12}$[/tex] V cubic inches

The volume of the cylindrical glass is given by:

[tex]$V_{\text{cylindrical glass}} = \pi r^2h$[/tex]

Here,

Radius r = 1 inch

Height h = 6 inches

[tex]$V_{\text{cylindrical glass}} = \pi r^2h = \pi (1)^2(6) = 6 \pi$[/tex]

Since the glass is half full of lemonade, the volume of lemonade in the glass is:

[tex]$V_{\text{lemonade}} = \frac{1}{2}V_{\text{cylindrical glass}} = \frac{1}{2} 6 \pi = 3\pi$[/tex]

The volume of lemon juice in the lemonade is given by:

[tex]$V_{\text{lemon juice}} = \frac{1}{12}V$[/tex]


Therefore

[tex]$V_{\text{lemon juice}} = \frac{1}{12}3\pi = \frac{1}{4}\pi = 0.7854$[/tex] cubic inches

Hence, the volume of lemon juice in the glass is 0.38 cubic inches (rounded to the nearest hundredth).

See more about volume at: https://brainly.com/question/29075763

#SPJ11

The perimeter of a rectangle is 3.5 times the length. The width is 2.5 cm less than the length. What is the width?

Answers

It is 7.5

the steps are in the photo

Solve the inequality for u.
48<5u+8
Simplify the answer as much as possible

Answers

Answer:

U=8

Step-by-step explanation:

48<5U+8

-5U<8-48

-5U<-40

-5U/-5<-40/+5

U>8

pls answert this within 1 hour or else i will be DOOMED

Answers

In the fractions prompts given, the correct output are:

(1 ÷2) ÷ 4 = 1/8(1 ÷5) ÷ 2 = 1/10(1 ÷3) ÷ 5 = 1/15(1 ÷4) ÷ 4 = 1/16The solution to the problem for (1/2) ÷ 3 is given below.

What is a fraction?

A fraction represents a part of a whole. It consists of a numerator and a denominator, with the numerator indicating the number of parts and the denominator indicating the total number of parts.

The calculations are given as follows;

1 )

= (1/2) x (1/4) [dividing by a number is the same as multiplying by its reciprocal]

= 1/8

2) (1/5) ÷ 2

= (1/5) x (1/2)

= 1/10

3)

(1/3) ÷ 5

= (1/3) x (1/5)

= 1/15

4) (1/4) ÷ 4

= (1/4) x (1/4)

= 1/16

5) One day, Amy baked a cake and wanted to divide it equally among 3 of her friends. She realized she only had half a cake left, so she decided to divide it into equal parts. Each friend received 1/6 of a cake. To check her calculation, she multiplied 1/6 by 3 and got 1/2. Thus, (1/2) ÷ 3 = 1/6, since dividing by 3 is the same as multiplying by 1/3.

Learn more about Fractions on:

https://brainly.com/question/20323470

#SPJ1

The null hypothesis is rejected whenever:A. past studies prove it wrong. B. there is a low probability that the obtained results could be due to random error.C. the independent variable fails to have an effect on the dependent variable.D. the researcher is convinced that the variable is ineffective in causing changes in behavior.

Answers

The null hypothesis is rejected whenever "there is a low probability that the obtained results could be due to random error." The correct answer is Option B.

What is the null hypothesis?

The null hypothesis is a statistical hypothesis used to test the difference between two sample data groups. The null hypothesis is the hypothesis that the sample statistics are not significantly different. Any significant differences between the sample data are seen as supporting the alternative hypothesis.

A null hypothesis is often expressed as "no difference," "no correlation," or "no significant effect." For example, the null hypothesis for an experiment comparing two groups of people could be "there is no difference between the two groups." When the null hypothesis is rejected, it means that the results of the experiment are statistically significant, and the alternative hypothesis is supported.

Therefore, the null hypothesis is rejected whenever there is a low probability that the obtained results could be due to random error.

Learn more about null hypothesis here: https://brainly.com/question/29576929

#SPJ11

what is 2 1/2 + x = 3 1/2. Please answer it quick

Answers



To solve for x in the equation 2 1/2 + x = 3 1/2, we can start by subtracting the mixed number on the left side from both sides of the equation. This gives us:

x = 3 1/2 - 2 1/2

Next, we can simplify each mixed number to an improper fraction and subtract them as follows:

x = (7/2) - (5/2)
x = (7-5)/2
x=1

Therefore, x equals one.

Answer:

x=1

Step-by-step explanation:

2.5+x=3.5

3.5-2.5=x

1=x

x=1

A ball is thrown straight up into the air. The x-axis shows the time, in seconds, and the y-axis shows the height of the ball, in feet, at any time (x)

Answers

Based on the information in the graph, it can be inferred that the false sentence is: The ball landed 11 feet from the person (option C).

How to identify the false option?

To identify the false option we must read all the options and check them with the information in the graph:

The first option is true because the highest point the ball reaches is between 11 and 12 feet high.The second option is true because the person drops the ball about 5 feet above the ground.The third option is false because the graph does not show the displacement of the ball with respect to the person who throws it. It only shows your height and the time elapsed in the launch.The fourth option is true because the ball falls between 0.7 and 1.45 seconds.

According to the above, the correct answer to this question would be option C because the sentence is false with respect to the information in the graph.

Learn more about Cartesian plane in: https://brainly.com/question/28574364

#SPJ1

The terminal ray of angle A, drawn in standard position, passes through the point (-4,
-6). What is the value of sec(A)?
Give your answer in simpliest radical form.

Answers

The value of sec A as required to be determined in the task content is; -√13 / 2.

What value represents sec A in the given scenario?

As evident from the task content; it follows that the terminal ray of angle A, drawn in standard position, passes through the point (-4, -6).

Therefore, the length that the line from the origin to A has length;

L = √((-4)² + (-6)²)

L = √52.

On this note, it follows that the value of sec A which is represented by; hypothenuse/ adjacent is;

sec (A) = -√52 / 4

sec (A) = -√13 / 2.

Ultimately, the value of sec (A) as required is; -√13 / 2.

Read more on trigonometric ratios;

https://brainly.com/question/10417664

#SPJ1

A triangle has side lengths of 6, 8, and 10. Is it a right triangle.
A. No, the sum of the legs is not equal to the hypotenuse.
B. Yes, the sum of the legs is equal to the hypotenuse.
C. No, the sum of the square of the legs is not equal to the square of the hypotenuse.
D. Yes, the sum of the square of the legs is equal to the square of the hypotenuse.
I know its a right angle so it's B or D, but which one.

Answers

Answer: D. Yes, the sum of the square of the legs is equal to the square of the hypotenuse.

Step-by-step explanation:

Pythagorean theorem is a^2 + b^2 = c^2

given side length of 6 and 8 and hypotenuse of 10

6^2 + 8^2 = 10^2

36 + 64 = 100

100 = 100

so yes, the sum of the square of the legs = the square of the hypotenuse.

1/1 point (graded) Compute X(), the matrix of predicted rankings UVT given the initial values for U() and V (0). 2 1 (Enter your answer as a matrix, e.g., type [[2,1],[1,0],[3,-1]] for a 3 x 2 matrix 1 0 Note the square brackets, and 3 -1 commas as separators. ) [[24,12,6], [0,0,0], (12,6,3], [24 ✓ 24 12 6 0 0 0 12 6 3 24 12 6

Answers

The matrix of predicted rankings UVT is [[48,24,12],[0,0,0],[24,12,6]].

The matrix of predicted rankings UVT can be calculated using the formula UVT = UV.The provided initial values for U() and V(0) are as follows:U() = [[2,1],[1,0],[3,-1]]V(0) = [[24,12,6],[0,0,0],[12,6,3]]Using the above values, the matrix of predicted rankings UVT can be computed as follows:UVT = UVU = [[2,1],[1,0],[3,-1]]V = [[24,12,6],[0,0,0],[12,6,3]]UVT = [[2,1],[1,0],[3,-1]] x [[24,12,6],[0,0,0],[12,6,3]]= [[48,24,12],[0,0,0],[24,12,6]]Therefore, the matrix of predicted rankings UVT is [[48,24,12],[0,0,0],[24,12,6]].

Learn more about Rankings

brainly.in/question/49424295

#SPJ11

Other Questions
the law prohibits anyone under the age of 18 from using any type of wireless device while driving, except in an emergency situation.True or false during his pea plant studies mendel found that the f2 generation produced a nearly 3:1 ratio of purple to white flowers.which principle explains why the ratio is not exactly 3:1 you calculate the value of a bond in excel with a yield of 10 percent. which change can you make in the calculations that will result in the value of the bond increasing? a tennis ball is dropped from 1.0 m, bounces off the ground, and rises to 0.85 m. what kind of collision occurred between the ball and the ground? Baroreceptors in the brain detect blood pressure, and osmoreceptors detect salt concentration in the blood. When changes occur, these receptors send signals to the pituitary to stimulate or inhibit antidiuretic hormone (ADH) release. If a man becomes lost in a desert without water, the receptors in his brain detect changes to his blood pressure and salt levels in his blood that signal the release of ADH into his bloodstream. Predict how the release of ADH will affect his kidneys and his blood vessels. A. Water reabsorption by kidneys and vasoconstriction of blood vessels B. Water reabsorption by kidneys and vasodilation of blood vessels C. Water excretion by kidneys and vasodilation of blood vessels D. Water excretion by kidneys and vasoconstriction of blood vessels what in seedless plants, a multicellular organ where meiosis occurs and haploid cells develop if the volume of a cube is 125 cm what is its surface area the per night cost for a room at a particular hotel is known to be uniformly distributed and range between 200 and 325. what is the average cost of a room at this hotel? (hint: use the uniform formula: mu TRUE/FALSE.if a company is conducting a panel interview, a job candidate should establish eye contact with only the person who asked the question. Help please! I have no idea!!!! True or False: The strength of the group dynamics of a team depends mostly on how willing leaders are to share authority, responsibility, information, and resources. 3 - cmo estn? label each drawing with a complete sentence, using the nouns provided with estar and the past participle of the verbs. Pavo / servir dormitorio / desordenar cama / hacer nias / dormir Currents tend to move in large ____ patterns in the Northern Hemisphere.clockwisecounterclockwiserandomirregular it takes 6 painters 4 1/2 to paint these classroom. calculate how long 3 painters will take to complete the same job Using the letters on the image, identify each component of the Bunsen burner.BACDA Choose...B Choose.... YC Choose...D Choose... what are top three businesses with the most franchises in the world? you know that a pair of triangles has two pairs of congruent angles. what other information do you need to show that the triangles are congruent?you need to know that _____ are congruent. any law passed to limit free trade between nations is called a trade what happens to the domains of an object when it becomes magnetized? Can someone help me find the elevation of the sun I need the answers that are highlighted in yellow please help image below